Pardon Our Interruption

As you were browsing something about your browser made us think you were a bot. There are a few reasons this might happen:

  • You've disabled JavaScript in your web browser.
  • You're a power user moving through this website with super-human speed.
  • You've disabled cookies in your web browser.
  • A third-party browser plugin, such as Ghostery or NoScript, is preventing JavaScript from running. Additional information is available in this support article .

To regain access, please make sure that cookies and JavaScript are enabled before reloading the page.

Are you seeking one-on-one college counseling and/or essay support? Limited spots are now available. Click here to learn more.

How to Write the AP Lang Synthesis Essay with Example

September 5, 2023

If you’re highly interested in learning more about writing analysis, then chances are you enrolled in AP Lang. Essentially, AP Lang is an advanced course for high schoolers that combines interest and knowledge in English with critical thinking. In the class, students learn how to analyze and synthesize a variety of texts to construct well-reasoned arguments. If you take AP Lang, then you can opt to take the AP test at the conclusion of the school year. On the exam, students write the AP Lang synthesis essay to demonstrate their learned abilities. In this article, we’ll look at what the AP Lang synthesis essay requires and show an example to provide better understanding of what to expect on the exam.

AP Lang Exam Basics

The AP Lang exam is separated into two sections. In the first section, students have one hour to answer a series of 45 multiple-choice questions. Here, about half of the questions are based on passages students read. The other half are focused on the best revision techniques. Essentially, the answers for the latter 20-22 questions are geared toward revising mock essays.

In this article, however, we’ll focus mainly on the second part of the exam: the AP Lang synthesis essay.

In this second section, students have two hours and 15 minutes to write three essays of their own design. The three open-ended questions in this section are intended to be free-response and allow for a variety of approaches. Each question is intended to allow up to 40 minutes to complete.

For the AP Lang synthesis essay, students are presented with a scenario of the College Board’s design. The scenario will provide its own thesis statement. Usually, scenarios relate to real-world problems like environmental concerns, media, or government policies.

For each scenario, students are provided with 6-7 outside sources. These sources could be in the form of an image, visual graph, or written paragraph. For written paragraphs, the sources are usually no more than 500 words.

Students are then expected to incorporate at least 3-4 of these outside sources into their essay response. The outside sources are intended to be used as supporting evidence for the student’s chosen stance or argument. Students are able to either agree with or disagree with the thesis presented in the original scenario.

AP Lang Exam – Scoring

In the second part of the AP Lang exam, students can earn a possible 6 points on each essay. 1 point is earned for the development of a thesis. Up to 4 points can be earned for evidence and commentary. The final 1 point is earned for sophistication of thought.

AP Lang Exam – Takeaways

Ultimately, the goal of the AP Lang synthesis essay is not whether the student is “right” or “wrong” in their argument. The key is that students are able to reasonably and clearly support their argument using the provided sources as evidence .

The College Board looks for your ability to identify relationships between texts , form a coherent argument , and interpret external sources .

Synthesis Essay AP Lang Examples

If you’re not sure how the questions will look on the AP Lang synthesis essay section, we’ll provide an example. After the example, we’ll break down the strengths and weaknesses of the response. That way, you’ll have a better idea of what the College Board is looking for.

Additionally, the College Board has released previous AP Lang synthesis essay examples you can review. They even have essay questions as recent as 2022 . For further support, a scoring commentary and comments from the Chief Reader are also available to view. Additionally, there are other examples you can view from earlier years .

Note: A good strategy to study for the synthesis essay AP Lang exam is to review your rhetorical devices and literary devices . Understanding how these devices function can be essential in constructing a cohesive essay.

Synthesis Essay AP Lang Examples – Sample Question

Below is a sample question from the AP Lang synthesis essay and a response to the prompt. This question was taken directly from a 2022 exam . However, the response to the question will be originally crafted for the purpose of this newsletter. As well, all supporting evidence will be originally created and does not correspond to any previous test.

The Question

Since the early 2000s, the United States government and a number of corporations have sponsored initiatives to improve education in the STEM disciplines: science, technology, engineering, and mathematics. The emphasis on STEM subjects in elementary, secondary, and higher education reflects concerns that United States students are less proficient in these areas than are students in other countries. Additionally, there is a belief that mastery in STEM fields is now essential in order to join a highly technical and specialized workforce. However, not everyone is convinced that a STEM-focused curriculum is necessary and/or effective.

In your response you should do the following:

  • Respond to the prompt with a thesis that presents a defensible
  • Select and use evidence from at least three of the provided sources to support your line of Indicate clearly the sources used through direct quotation, paraphrase, or summary. Sources may be cited as Source A, Source B, etc., or by using the description in parentheses.
  • Explain how the evidence supports your line of
  • Use appropriate grammar and punctuation in communicating your

How to Approach the Question

Maybe your first thought upon seeing this block of text is to feel overwhelmed. But don’t panic. There are effective ways to approach the question so you will be more prepared in your response.

It’s a good strategy to first isolate the thesis . What is the main idea of the text, and what is its argument?

Try it out. Reread the prompt and see if you can identify what the statement is asking you to develop an opinion on.

Think you’ve got it? In this example, we will be focusing on whether or not a STEM-focused curriculum in K-12 education is necessary and/or effective. In short, we will be arguing either for (highlighting the benefits) or against (highlighting the pitfalls) a STEM-focused curriculum.

How do we know what this statement is asking us?

Well, the statement provides a lot of background information. For example, we receive a definition of what STEM stands for. As well, we know that since 2000, there has been a greater initiative for STEM-focused classes.

When you read the prompt for the first time, it’s a great strategy to learn how to differentiate between background and contextual information from the heart of the argument .

A good way to learn how to isolate the argument is to look for transition words. Usually, these appear near the end of the question. Words like “however” and “yet” are signals that the statement is offering a differing opinion. Typically, the statement will tell you which two positions it’s offering for argument. These opinions are usually signaled by contrasting transition words.

So, now that we know what the question is asking us, what is the best way to respond?

Synthesis Essay AP Lang Examples – Sample Answer

The following is an essay response I crafted to the above question. After reading the sample, I will break down what it does well and what areas can be improved.

A STEM-focused curriculum is not as essential to providing a meaningful K-12 education. Because the majority of high school students are not proficient in STEM-focused classes, prioritizing these classes causes harm to student’s mental health and academic performance.

As seen in Source A, 60% of high school seniors in the Midwest only scored a C average in math and science-based classes (Langston). This statistic suggests that the majority of students do not resonate with STEM classes and therefore perform poorly. Earning a low score in any class does not bode well for students’ mental health.

When looking at the primary argument in Source C, it’s clear that most high schoolers prefer creative outlets to fact-based research (Kohler). Allowing students the opportunity to be more creative and initiate conversations about coursework lets students be more active in their learning. When students can discuss the nuance in their opinions, more personal growth happens. These conversations are not always easy to have in STEM-focused classes.

As well, when looking back to Source A, it’s clear that high school students in the Midwest earned higher grades, on average, in their English and art classes (Langston). This figure suggests that students perform better in these classes because they relate more to the source material. When relating to what they learn, they perform better in class.

In conclusion, STEM-focused curriculum is not as essential in K-12 education because most high school students do not relate to their STEM classes. When students do not earn satisfactory grades in these classes, it negatively affects their future college applications and job prospects.

Synthesis Essay AP Lang Examples – Answer Breakdown

So, what does this essay response get right, and where can it be improved? Let’s start with what the response does well.

First, the response establishes its thesis right away. Usually, it’s a good idea to clearly state your argument within the first paragraph. Not only is this a good practice because a reader can easily identify your stance, but also you can refer to your thesis as you write to make you stay on track.

With your thesis, it’s also a good idea to include one to two supporting sentences with the reasons why the thesis is concluded . Like in this example, I wrote that STEM-focused classes should not be prioritized because they can negatively affect both mental health and academic performance.

Another positive aspect of this response is that it is sure to not only reference but also cites its sources . It’s important that the reader understand where your information is coming from. That way, the readers can ensure you are interpreting the sources correctly.

AP Lang Synthesis Essay (Continued)

However, when rereading the instructions, it’s clear that this response fails the basic requirement of referring to at least three sources. Always make sure to reread the instructions to ensure you meet the standard requirements for incorporating source material.

Further, this AP Lang synthesis essay does not fully support its arguments . Ideas are simply stated and are not expanded upon.

For example, I mentioned a few times that earning low grades in STEM classes leads to negative mental health for high school students. However, there is no source referenced that either confirms or denies this claim. Therefore, there is no sufficient evidence to support my argument. It relies purely on inference.

Additionally, this AP Lang synthesis essay does not arrive at a sufficient level of sophistication of thought . Basically, sophistication of thought means avoiding broad generalizations and vague claims. The more specific you can be, the better your argument will sound.

Synthesis Essay AP Lang – In Conclusion

In the end, it’s always helpful to read the prompt thoroughly before writing. As well, making notes while you read could be a good strategy to pinpoint main ideas both in the prompt and the sources. That way, you can reread the material quickly. Similarly, sketching an outline may also be helpful. In addition, you should always carefully read the instructions to ensure all guidelines are followed.

As long as you avoid broad generalizations and use enough supporting evidence for your claim, you will be on the right path!

  • High School Success

Meghan Dairaghi

With a BA in English and an MFA in Creative Writing, Meghan has served as a writing tutor at the University of Missouri St. Louis and Maryville University. Additionally, Meghan has held editorial roles at River Styx and Boulevard, and was a prose reader at Farside Review . Most recently, her work has been featured in Belle Ombre , Flypaper Lit , and Mag 20/20 , among others, and she was nominated for the Mary Troy Prize in Fiction. 

  • 2-Year Colleges
  • ADHD/LD/Autism/Executive Functioning
  • Application Strategies
  • Best Colleges by Major
  • Best Colleges by State
  • Big Picture
  • Career & Personality Assessment
  • College Essay
  • College Search/Knowledge
  • College Success
  • Costs & Financial Aid
  • Data Visualizations
  • Dental School Admissions
  • Extracurricular Activities
  • Graduate School Admissions
  • High Schools
  • Homeschool Resources
  • Law School Admissions
  • Medical School Admissions
  • Navigating the Admissions Process
  • Online Learning
  • Outdoor Adventure
  • Private High School Spotlight
  • Research Programs
  • Summer Program Spotlight
  • Summer Programs
  • Teacher Tools
  • Test Prep Provider Spotlight

“Innovative and invaluable…use this book as your college lifeline.”

— Lynn O'Shaughnessy

Nationally Recognized College Expert

College Planning in Your Inbox

Join our information-packed monthly newsletter.

PrepScholar

Choose Your Test

  • Search Blogs By Category
  • College Admissions
  • AP and IB Exams
  • GPA and Coursework

Expert Guide to the AP Language and Composition Exam

author image

Advanced Placement (AP)

article-71342_640.jpg

With the 2023 AP English Language and Composition exam happening on Tuesday, May 9, it's time to make sure that you're familiar with all aspects of the exam. In this article, I'll give a brief overview of the test, do a deeper dive on each of the sections, discuss how the exam is scored, offer some strategies for studying, and finally wrap up with some essential exam day tips.

Exam Overview

The AP Language and Composition exam tests your rhetorical and composition skills. Essentially, how do authors construct effective arguments in their writing? What tools do they use? How can you use those tools to craft effective writing yourself? That is the essence of rhetorical analysis.

The exam has two parts: the first section is an hour-long, 45 question multiple-choice section. It includes five sets of questions, each based on a passage or passages. In this section, there will be 23-25 rhetorical analysis questions which test your rhetorical skills. There will also be 20-22 writing questions which require you to consider revisions to the texts you're shown.

The second section is free response. It starts with a 15-minute reading period, and then you'll have 120 minutes to write three analytical essays:

  • One essay where you synthesize several provided texts to create an argument
  • One essay where you analyze a nonfiction passage for its rhetorical construction
  • One essay where you create an original argument in response to a prompt.

You will have about 40 minutes to write each essay, but no one will prompt you to move from essay to essay—you can structure the 120 minutes as you wish.

In the next sections I'll go over each section of the exam more closely—first multiple choice, and then free response.

The AP English Language and Composition Multiple-Choice

The multiple-choice section tests you on two main areas. The first is how well you can read and understand nonfiction passages for their use of rhetorical devices and tools. The second is how well you can "think like a writer" and make revisions to texts in composition questions.

You will be presented with five passages, about which you will receive a small amount of orienting information, e.g. "This passage is excerpted from a collection of essays on boating" or "This passage is excerpted from an essay written in 19th-century Haiti." Each passage will be followed by a set of questions.

There are, in general, eight question types you can expect to encounter on the multiple-choice section of the exam. I've taken my examples from the sample questions in the " Course and Exam Description ."

eight-1316133_640.jpg

Magic eight-ball says there are eight types of multiple-choice questions!

Type 1: Reading Comprehension

These questions are focused on verifying that you understood what a certain part of the passage was saying on a concrete, literal level. You can identify these questions from phrases like "according to" "refers," etc. The best way to succeed on these questions is to go back and re-read the part of the passage referred to very carefully.

Comprehension.png

Type 2: Implication

These questions take reading comprehension one step further—they are primarily focused on what the author is implying without directly coming out and saying it. These questions will have a correct answer, though, based on evidence from the passage. Which interpretation offered in the answers does the passage most support? You can identify questions like these from words like "best supported," ‘"implies," "suggests," "inferred," and so on.

implies.png

Type 3: Overall Passage and Author Questions

These questions ask about overall elements of the passage or the author, such as the author's attitude on the issue discussed, the purpose of the passage, the passage's overarching style, the audience for the passage, and so on.

You can identify these questions because they won't refer back to a specific moment in the text. For these questions, you'll need to think of the passage from a "bird's-eye view" and consider what all of the small details together are combining to say.

3overall_passage.png

Type 4: Relationships Between Parts of the Text

Some questions will ask you to describe the relationship between two parts of the text, whether they are paragraphs or specific lines. You can identify these because they will usually explicitly ask about the relationship between two identified parts of the text, although sometimes they will instead ask about a relationship implicitly, by saying something like "compared to the rest of the passage."

4relationship.png

Type 5: Interpretation of Imagery/Figurative Language

These questions will ask you about the deeper meaning or implication of figurative language or imagery that is used in the text. Essentially, why did the author choose to use this simile or this metaphor? What is s/he trying to accomplish?

You can generally identify questions like this because the question will specifically reference a moment of figurative language in the text. However, it might not be immediately apparent that the phrase being referenced is figurative, so you may need to go back and look at it in the passage to be sure of what kind of question you are facing.

5imagery.png

Type 6: Purpose of Part of the Text

Still other questions will ask you to identify what purpose a particular part of the text serves in the author's larger argument. What is the author trying to accomplish with the particular moment in the text identified in the question?

You can identify these questions because they will generally explicitly ask what purpose a certain part of the text serves. You may also see words or phrases like "serves to" or "function."

6purpose_of_part.png

Type 7: Rhetorical Strategy

These questions will ask you to identify a rhetorical strategy used by the author. They will often specifically use the phrase "rhetorical strategy," although sometimes you will be able to identify them instead through the answer choices, which offer different rhetorical strategies as possibilities.

7rhetorical_strategy.png

Type 8: Composition

This is the newest question type, first seen in the 2019/2020 school year. For these questions, the student will need to act as though they are the writer and think through different choices writers need to make when writing or revising text.

These questions can involve changing the order of sentences or paragraphs, adding or omitting information to strengthen an argument or improve clarity, making changes to draw reader attention, and other composition-based choices.

body_composition

Some very important stylish effects going on here.

The AP English Language and Composition Free Response

The free response section has a 15-minute reading period. After that time, you will have 120 minutes to write three essays that address three distinct tasks.

Because the first essay involves reading sources, it is suggested that you use the entire 15-minute reading period to read the sources and plan the first essay. However, you may want to glance at the other questions during the reading period so that ideas can percolate in the back of your mind as you work on the first essay.

Essay One: Synthesis

For this essay, you will be briefly oriented on an issue and then given anywhere from six to seven sources that provide various perspectives and information on the issue. You will then need to write an argumentative essay with support from the documents.

If this sounds a lot like a DBQ , as on the history AP exams, that's because it is! However, this essay is much more argumentative in nature—your goal is to persuade, not merely interpret the documents.

Example (documents not included, see 2022 free response questions ):

body-AP-Literature-synthesis

Essay Two: Rhetorical Analysis

In the second essay, you'll be presented with an excerpt from a nonfiction piece that advances an argument and asked to write an essay analyzing the rhetorical strategies used to construct the passage's argument. You will also be given some orienting information—where the passage was excerpted from, who wrote it, its approximate date, where it was published (if at all), and to whom it was directed.

Example (excerpt not included, see 2022 free response questions ):

body-AP-literature-Question-2

Essay Three: Argument

In the third essay, you will be presented with an issue and asked to write a persuasive essay taking a position on the issue. You will need to support your position with evidence from your "reading, experience, and observations."

body-AP-lit-Question-3

This doesn't look like a very well-constructed argument.

How The AP Language and Composition Exam Is Scored

The multiple-choice section of the exam is worth 45% of your score, and the free-response section is worth the other 55%. So each of the three free-response essays is worth about 18% of your score.

As on other APs, your raw score will be converted to a scaled score of 1-5. This exam has a relatively low 5 rate. Only 10% of test takers received a 5 in 2022 , although 56% of students received a score of 3 or higher.

In terms of how the raw score is obtained, the multiple-choice section is similar to other AP multiple-choice sections: you receive a point for every question you answer correctly, and there is no penalty for guessing.

The grading rubrics for the free-response questions were revamped in 2019. They are scored using analytic rubrics instead of holistic rubrics. For each free-response question, you will be given a score from 0-6. The rubrics assess three major areas:

#1: Thesis (0 to 1 points): Is there a thesis, and does it properly respond to the prompt?

#2: Evidence and Commentary (0 to 4 points): Does the essay include supporting evidence and analysis that is relevant, specific, well organized, and supports the thesis?

#3: Sophistication (0 to 1 points): Is the essay well-crafted and does it show a sufficiently nuanced understanding of the prompt?

Each scoring rubric broadly assesses these three factors. However, each task is also different in nature, so the rubrics do have some differences. I'll go over each rubric—and what it really means—for you here.

Synthesis Essay Rubrics

0 For any of the following:
1

EVIDENCE AND COMMENTARY

0
1 AND
2 AND
3 AND
4 AND

SOPHISTICATION

0
1 Responses that earn this point may demonstrate sophistication of thought and/or a complex understanding of the rhetorical situation by doing any of the following:

dough-196235_640.jpg

Time to synthesize this dough into some cookies.

Rhetorical Analysis Essay Rubrics

0
1 AND
2 AND
3 AND AND
4 AND AND

magnifying-glass-914922_640.png

Examine your texts closely!

Argumentative Essay Rubrics

playing-puppies-790638_640.jpg

The best kind of frenzy is a puppy frenzy!

AP English Language Prep Tips

Unlike its cousin, the AP English Literature and Composition exam, the AP Language and Composition exam (and course) have very little to do with fiction or poetry. So some students used to more traditional English classes may be somewhat at a loss as to what to do to prepare.

Luckily for you, I have a whole slate of preparation tips for you!

Read Nonfiction—In a Smart Way

A major thing you can do to prepare for the AP Lang and Comp exam is to read nonfiction— particularly nonfiction that argues a position , whether explicitly (like an op-ed) or implicitly (like many memoirs and personal essays). Read a variety of non-fiction genres and topics, and pay attention to the following:

  • What is the author's argument?
  • What evidence do they use to support their position?
  • What rhetorical techniques and strategies do they use to build their argument?
  • Are they persuasive? What counterarguments can you identify? Do they address them?

Thinking about these questions with all the reading you do will help you hone your rhetorical analysis skills.

Learn Rhetorical Terms and Strategies

Of course, if you're going to be analyzing the nonfiction works you read for their rhetorical techniques and strategies, you need to know what those are! You should learn a robust stable of rhetorical terms from your teacher, but here's my guide to the most important AP Language and Composition terms .

  • We've compiled a list of 20 rhetorical devices you should know.
  • A heroic individual from Riverside schools in Ohio uploaded this aggressively comprehensive list of rhetorical terms with examples. It's 27 pages long, and you definitely shouldn't expect to know all of these for the exam, but it's a useful resource for learning some new terms.
  • Another great resource for learning about rhetorical analysis and how rhetorical devices are actually used is the YouTube Channel Teach Argument , which has videos rhetorically analyzing everything from Taylor Swift music videos to Super Bowl commercials. It's a fun way to think about rhetorical devices and get familiar with argumentative structures.
  • Finally, a great book—which you might already use in your class—is " They Say, I Say. " This book provides an overview of rhetoric specifically for academic purposes, which will serve you well for AP preparation and beyond.

You also need to practice argumentative and persuasive writing. In particular, you should practice the writing styles that will be tested on the exam: synthesizing your own argument based on multiple outside sources, rhetorically analyzing another piece of writing in-depth, and creating a completely original argument based on your own evidence and experience.

You should be doing lots of writing assignments in your AP class to prepare, but thoughtful, additional writing will help. You don't necessarily need to turn all of the practice writing you do into polished pieces, either—just writing for yourself, while trying to address some of these tasks, will give you a low-pressure way to try out different rhetorical structures and argumentative moves, as well as practicing things like organization and developing your own writing style.

once-upon-a-time-719174_640.jpg

Not the most auspicious start to an argumentative essay.

Practice for the Exam

Finally, you'll need to practice specifically for the exam format. There are sample multiple-choice questions in the " AP Course and Exam Description ," and old free-response questions on the College Board website.

Unfortunately, the College Board hasn't officially released any complete exams from previous years for the AP English Language and Composition exam, but you might be able to find some that teachers have uploaded to school websites and so on by Googling "AP Language complete released exams." I also have a guide to AP Language and Composition practice tests .

Once you're prepped and ready to go, how can you do your best on the test?

Looking for help studying for your AP exam? Our one-on-one online AP tutoring services can help you prepare for your AP exams. Get matched with a top tutor who got a high score on the exam you're studying for!

AP Language and Composition Test Day Tips

Here are four key tips for test-day success.

board-1193334_640.jpg

You are one hundred percent success!

Interact With the Text

When you are reading passages, both on the multiple-choice section and for the first two free-response questions, interact with the text! Mark it up for things that seem important, devices you notice, the author's argument, and anything else that seems important to the rhetorical construction of the text. This will help you engage with the text and make it easier to answer questions or write an essay about the passage.

Think About Every Text's Overarching Purpose and Argument

Similarly, with every passage you read, consider the author's overarching purpose and argument. If you can confidently figure out what the author's primary assertion is, it will be easier to trace how all of the other aspects of the text play into the author's main point.

Plan Your Essays

The single most important thing you can do for yourself on the free-response section of the AP English Language exam is to spend a few minutes planning and outlining your essays before you start to write them.

Unlike on some other exams, where the content is the most important aspect of the essay, on the AP Language Exam, organization, a well-developed argument, and strong evidence are all critical to strong essay scores. An outline will help you with all of these things. You'll be able to make sure each part of your argument is logical, has sufficient evidence, and that your paragraphs are arranged in a way that is clear and flows well.

Anticipate and Address Counterarguments

Another thing you can do to give your free responses an extra boost is to identify counterarguments to your position and address them within your essay. This not only helps shore up your own position, but it's also a fairly sophisticated move in a timed essay that will win you kudos with AP graders.

envelope-392962_640.jpg

Address counterarguments properly or they might get returned to sender!

Key Takeaways

The AP Language and Composition exam tests your rhetorical skills. The exam has two sections.

The first section is an hour-long, 45 question multiple-choice test based on the rhetorical techniques and composition choices.

The second section is a two-hour free-response section (with a 15-minute initial reading period) with three essay questions: one where you must synthesize given sources to make an original argument, one where you must rhetorically analyze a given passage, and one where you must create a wholly original argument about an issue with no outside sources given.

You'll receive one point for every correct answer on the multiple-choice section of the exam, which is worth 45% of your score. The free-response section is worth 55% of your score. For each free-response question, you'll get a score based on a rubric from 0-6. Your total raw score will be converted to a scaled score from 1-5.

Here are some test prep strategies for AP Lang:

#1 : Read nonfiction with an eye for rhetoric #2 : Learn rhetorical strategies and techniques #3 : Practice writing to deploy rhetorical skills #4 : Practice for the exam!

Here are some test-day success tips:

#1 : Interact with each passage you encounter! #2 : Consider every text's overarching purpose and argument. #3 : Keep track of time #4 : Plan your essays #5 : Identify and address counterarguments in your essays.

With all of this knowledge, you're ready to slay the AP English Language and Composition beast!

animal-1299749_640.png

Noble knight, prepare to slay the AP dragon!

What's Next?

Want more AP Lang review? We have a complete collection of released AP Language practice tests , as well as a list of the AP Lang terms you need to know and a guide to the multiple choice section .

Taking the AP Literature exam? Check out our ultimate guide to the AP English Literature test and our list of AP Literature practice tests .

Taking other AP exams? See our Ultimate Guides to AP World History , AP US History , AP Chemistry , AP Biology , AP World History , and AP Human Geography .

Need more AP prep guidance? Check out how to study for AP exams and how to find AP practice tests .

Want to build the best possible college application?   We can help.   PrepScholar Admissions combines world-class admissions counselors with our data-driven, proprietary admissions strategies. We've guided thousands of students to get into their top choice schools, from state colleges to the Ivy League. We know what kinds of students colleges want to admit and are driven to get you admitted to your dream schools. Learn more about PrepScholar Admissions to maximize your chance of getting in:

These recommendations are based solely on our knowledge and experience. If you purchase an item through one of our links, PrepScholar may receive a commission.

Trending Now

How to Get Into Harvard and the Ivy League

How to Get a Perfect 4.0 GPA

How to Write an Amazing College Essay

What Exactly Are Colleges Looking For?

ACT vs. SAT: Which Test Should You Take?

When should you take the SAT or ACT?

Get Your Free

PrepScholar

Find Your Target SAT Score

Free Complete Official SAT Practice Tests

How to Get a Perfect SAT Score, by an Expert Full Scorer

Score 800 on SAT Math

Score 800 on SAT Reading and Writing

How to Improve Your Low SAT Score

Score 600 on SAT Math

Score 600 on SAT Reading and Writing

Find Your Target ACT Score

Complete Official Free ACT Practice Tests

How to Get a Perfect ACT Score, by a 36 Full Scorer

Get a 36 on ACT English

Get a 36 on ACT Math

Get a 36 on ACT Reading

Get a 36 on ACT Science

How to Improve Your Low ACT Score

Get a 24 on ACT English

Get a 24 on ACT Math

Get a 24 on ACT Reading

Get a 24 on ACT Science

Stay Informed

Get the latest articles and test prep tips!

Follow us on Facebook (icon)

Ellen has extensive education mentorship experience and is deeply committed to helping students succeed in all areas of life. She received a BA from Harvard in Folklore and Mythology and is currently pursuing graduate studies at Columbia University.

Ask a Question Below

Have any questions about this article or other topics? Ask below and we'll reply!

Tackling the AP English Language and Composition Essays: Part 3

Statistical Mediation & Moderation in Psychological Research (1)-3

As you may recall from Part 1 , the Synthesis and Argument essays require that you take an argumentative position on a topic. In the Synthesis Essay, you are given a topic or question (e.g., is the death penalty just?) and six sources addressing that topic or question. You will then be asked to take an argumentative position, citing at least three of the sources. In the Argument Essay, you are given a topic or idea typically in the form of an author’s quote: for example, a quote from a famous writer saying that, in modern society, it’s necessary to be a little bit unethical to succeed. You are then asked to take an argumentative position on this topic or idea. This time, though, there will be no sources for you to cite. All of your thinking about the topic must come from your own head.

We’ve already discussed the Rhetoric Essay, and how to go through the process of writing a good one. Much of the same advice still applies to these two essays! Here are the things that, when it comes the Synthesis and Argument Essay, remain the same:

  • The grading rubric for the Synthesis and Argument Essays is the same as that for the Rhetoric Essay, awarding one point for thesis, four for evidence and analysis, and one for “sophistication.”
  • For all three essays, you must make a clear argument, supply sufficient evidence, and explain that evidence well. 
  • The six step process will also be the same for all three essays. Remember that the six steps are: (1) Organizing Your Time, (2) Reading and Annotating, (3) Outlining the thesis (4) Outlining the structure (5) Writing [paragraphs, evidence, analysis] (6) Writing [sentences].

But, lots of things are different, too. So, you need to go through the six steps of the process in a slightly different way for each essay...

The Synthesis Essay: Six Steps

1. organizing your time.

As discussed in Part 1 , the first 45 minutes of the essay section (including both the “reading” and “writing” periods) should be devoted to the Synthesis Essay. During the first 15 minutes, which are the designated “reading period,” you should read the question and six sources, and begin to brainstorm and outline your essay. As the “writing period” opens—the first thirty minutes of which you will now devote to the Synthesis Essay—you should be finishing your outline, and beginning to write. You should then write for about 25, reserving 5 minutes to proofread.

2. Reading and Annotating

As you read the six sources, keep a running list of the pro or con arguments that you encounter, and the corresponding evidence. I suggest you take a piece of paper and divide it into two columns, pro and con. As you read through the sources, you can use the columns to keep track of arguments and evidence. At the same time, you should underline particularly compelling pieces of evidence supplied for the arguments you’re recording, like statistics or particularly persuasive turns of phrase. 

As you proceed, you may start to feel like you know which side you want to defend. Great! At that point, I would start paying closer attention to the evidence supplied for arguments for that side, carefully noting which quotes or details you might cite. That being said, I would not stop recording the arguments for the other side. These are going to be the types of counter-positions you will need to answer in your essay.

By the time you’re done taking notes, you should have a good list of the arguments available for defending each side, as well as some evidence for those arguments. Now, you can use that material to make an outline.

3. Outlining Part 1: Thesis

Go look at the AP Lang test grading rubric (printed at the very end of this guide ), and look at the sample excellent theses that graders are given as models for earning the 1 thesis point. They all clearly take a stance on one side or another of the issue. This isn’t the time to be wishy-washy or even-handed. It’s a time to be decisive, and clearly take a side. Your thesis may address both sides of the issue, but only in the process of clearly deciding on one of those sides. It may for example, read something like: “Though use of public libraries will, in the digital age, no doubt dwindle, they remain essential to the future of our nation, and should be maintained.” It should not read like this: “Use of public libraries will start to dwindle in the digital age, but they do still seem in many ways essential.” In the first case, you acknowledge both sides, but it’s clear which side you stand on: this is the best strategy to use for this essay.

4. Outlining Part 2: Structure

Many different types of structures have earned perfect scores for the Synthesis Essay. That being said, there are a few guidelines to follow. 

First, you need to have multiple body paragraphs (aim for three, though two can be ok if they are robust). Each of those paragraphs needs to make one clear point, and each of those paragraphs’ points needs to be stated clearly and completely in an initial topic sentence. 

There are also some new rules to follow here. You must give at least two strong reasons for why your side is correct. And you must, in some way, address counterarguments: likely rebuttals of your points, or strong arguments for the other side.

There are many different structures you might use to do both of these things, and which structure you choose will depend on the arguments you want to make, as well as the strongest arguments the other side might marshal. Here are some possible structures that have all worked in perfectly scored essays:

  • Provide three different reasons why your side is right, each with its own paragraph. Address any potential counterarguments or concerns about these reasons, where necessary, within the paragraphs. 
  • Provide two reasons why your side is right, in two paragraphs. In a third paragraph, address and answer the strongest argument or case for the other side.

5. Writing Part 1: Paragraphs, Evidence, Analysis

When you go to write, make sure:

  • Each paragraph has a topic sentence.
  • Each paragraph supplies evidence to support that topic sentence’s argument.
  • The evidence is analyzed. For more information on any of these points, see Part 2 of this series.

There is one extra thing, though: you have to cite your sources. There are two ways to cite. First, you can simply make a point that one of the readings made, without quoting. Or, you can quote! I suggest that you quote directly at least once. It ensures that you are citing in detail, and you want to be citing and engaging with the arguments in detail. Be sure to put “Source #” or the author name in parentheses after your paraphrase or quote in order to cite the source.

6. Writing Part 2: Sentence by Sentence

Proofread your sentences, in the last few minutes. All the same principles apply ( see Part 2 for details ).

The Argument Essay: Six Steps

1-2. managing your time + brainstorming.

You have 45 minutes. But this essay should take you a bit less time than the other two, since it gives you no passages to read. Spend 5-7 minutes reading the question and then brainstorming/outlining. Then, spend the rest of the time writing, with some time at the end for proofreading. If you finish this essay before your 45 minutes are up, use your extra time to proofread everything you’ve written (all three essays). 

3-4. Outlining: Thesis and Structure

The same rules for thesis and structure from the Synthesis Essay apply to the Argument Essay. You need to take a clear position on the topic, even if you acknowledge the other side. And you need to provide multiple reasons for that position, while also addressing counterarguments. 

The difference, here, comes in the types of reasons you are going to give for your argument. In the Synthesis Essay, you were giving reasons largely supplied from the sources you read. Here, you have to come up with your own reasons and examples for them. In general, your reasons and examples are going to come from a few sources: 

  • Your personal life and experience: you might tell a story from your own life to show why the principle you’re defending is true.
  • Your knowledge of history, literature, or other nonfiction or school subjects: you might use an example of an event from a novel to support the point, or an anecdote from Renaissance history that you learned in school, or a tidbit from the biography of some inspiring person, like MLK or Marie Curie. 
  • Your knowledge of current events: you might be able to supply a reason for your point that refers to some major, hot button issue of current events, like climate change, or Black Lives Matter, or the benefits and downsides of social media. 
  • Your knowledge of other things: Know a lot about music? Or gardening? Or religion? Use examples from those domains.

What you might be gathering is that, in order to answer this essay question well, supplying arguments for general principles like “Money can’t buy happiness” or “quitters never win,” you need to know about…things. And as a teenager, you’ve only had so much time to get to know things. So, here’s what I advise: in preparation of the exam, make a list of 20 things that you are going to know about. Yes, you can choose common school topics like “The American Revolution” or The Scarlet Letter. But supplement those topics with the academic or political topics that you are actually interested in or passionate about, like the 1980s AIDS epidemic and struggle for LGBQT rights, or ancient Greek military history, or novels by Jane Austen or Toni Morrison, or the biography of an Olympic athlete. Passion speaks! Also, include some important or pivotal anecdotes from your life, especially times when you learned lessons. Once you have your list, make sure you actually know about these things. Know some details: dates, names, precise events, anecdotes, etc. Though you can’t predict what your Argument Essay question will be, having detailed knowledge of various topics will certainly help you out. 

5. Writing: Paragraphs (Evidence, Analysis)

As with the other essays, you still need topic sentences summarizing each paragraph’s argument (a reason for why your position is correct, drawing on some major example from history, culture, or personal life). You still need evidence for your argument (coming from your store of knowledge about…things). And you still need to analyze that evidence and explain why it proves your points, addressing counterarguments where possible. 

Only one more tip: when you supply your support, be detailed. Don’t refer to some event in your life in vague terms (“at first I found math hard, but then I practiced and got better”). Supply the details (“At first I found math hard, and in the first weeks of the term, I failed a test. Soon, however, I started to practice, drilling problems at my kitchen table every night, and asking my older brother to check over my work. By the end of the semester, I had greatly improved, and earned a B+ on the final exam.”). The same goes for historical or literary anecdotes: cite specific dates, names, events. In reading and writing, we call this “the reality effect”: an argument or story feels more real and more persuasive, when it includes little, hyper-specific details. “I ate some food,” doesn’t feel as real as, “Sitting on a bench in a park in West Palm Beach, I ate a banana and seven almonds.”

6. Writing: Sentence by Sentence

Proofread, at the end! The same tips from Part 2 apply.

Well, that just about covers the three AP Lang Essays! I have just two parting tips. First, practice, practice, practice. It will not feel easy writing three good essays at this pace the first time you do it (or the second, or the third). It will start to feel ok after many iterations, and that’s where you want to be. You don’t want your first, or even second time trying this out to be on test day. Second, try to have fun with it. Don’t go crazy, but psych yourself into getting into your essays. The readers can feel your enthusiasm. And they like it. Best of luck!

The   AP Exams are an opportunity for you to deepen your academic engagement, demonstrate your readiness for college learning, and earn college credit .  Our team of PhD candidates, composed of expert tutors like Tess, loves teaching AP material and helping students hone test taking strategies. Looking for customized AP support?

Contact us!

Related Content

This site uses various technologies, as described in our Privacy Policy, for personalization, measuring website use/performance, and targeted advertising, which may include storing and sharing information about your site visit with third parties. By continuing to use this website you consent to our Privacy Policy and Terms of Use .

We are experiencing sporadically slow performance in our online tools, which you may notice when working in your dashboard. Our team is fully engaged and actively working to improve your online experience. If you are experiencing a connectivity issue, we recommend you try again in 10-15 minutes. We will update this space when the issue is resolved.

Enter your email to unlock an extra $25 off an SAT or ACT program!

By submitting my email address. i certify that i am 13 years of age or older, agree to recieve marketing email messages from the princeton review, and agree to terms of use., guide to the ap english language and composition exam.

AP English Language Exam

Can you apply the rhetorical triangle to a piece of writing? Are you able to argue a position? The AP ® English Language and Composition exam tests topics and skills discussed in your Advanced Placement English Language course. If you score high enough, your AP English Language score could earn you college credit!

Check out our AP English Language Guide for what you need to know about the exam:

  • Exam Overview
  • Sections and Question Types
  • How to Prepare

What’s on the AP English Language & Composition Exam?

The College Board is very detailed in what they require your AP teacher to cover in his or her AP English Language & Composition course. The exam tests your abilities to understand how authors use rhetoric and language to convey their purpose. Students are also expected to apply these techniques to their own writing and research projects. Some of the major skills tested include the ability to:

  • Identify an author’s purpose and intended audience
  • Recognize rhetorical devices and strategies in an author’s work
  • Demonstrate understanding of citations in research papers
  • Apply these skills and techniques to their own writing
  • Create and organize an argument defended with evidence and reasoning
  • Plan, write, and revise cogent, well-written essays

Check out our line of AP guides  for a comprehensive content review.

AP English Language Sections & Question Types

The AP English Language & Composition exam is 3 hours and 15 minutes long and consists of two sections: a multiple-choice section and a free response section.

Section 1

60 minutes

45 multiple choice questions

45%

Section 2

2 hours and 15 minutes

3 free response questions

55%

Read More: Review for the exam with our  AP English Language Crash Course 

Multiple-Choice

For AP English Language multiple-choice questions, you are presented with two Reading Passages and three Writing passages. The two Reading passages are nonfiction passages taken from all sorts of works. The idea is to get you to focus on rhetorical devices, figures of speech and intended purposes, under rigid time constraints and with material you haven’t seen before. The three Writing passages are student-produced essays. The idea is to get you to revise the essay that help the writer accomplish his or her goal.

Free Response

The AP English Language section contains three essay prompts: a synthesis essay, a rhetorical analysis essay, and an argument essay.

  • Synthesis essay: You’ll be given a scenario and tasked with writing a response using at least three of six or seven short accompanying sources for support.
  • Rhetorical analysis essay: Asks you to analyze the techniques an author uses, and discuss how they contribute to the author’s purpose.
  • Argument essay: Presents a claim or assertion in the prompt and then asks you to argue a position based on your own knowledge, experience, or reading.

How to Interpret AP English Language Scores

AP scores are reported from 1 to 5. Colleges are generally looking for a 4 or 5 on the AP English Language exam, but some may grant AP credit for a 3. Each test is curved so scores vary from year to year. Here’s how AP English Lang students scored on the May 2022 test:

5

Extremely qualified

10.4%

4

Well qualified

21.1%

3

Qualified

24.2%

2

Possibly qualified

29.8%

1

No recommendation

14.5%

Source: College Board

How can I prepare?

AP classes are great, but for many students they’re not enough! For a thorough review of AP English Language content and strategy, pick the AP prep option that works best for your goals and learning style.

  • AP Exams  

Explore Colleges For You

Explore Colleges For You

Connect with our featured colleges to find schools that both match your interests and are looking for students like you.

Career Quiz

Career Quiz

Take our short quiz to learn which is the right career for you.

Connect With College Coaches

Get Started on Athletic Scholarships & Recruiting!

Join athletes who were discovered, recruited & often received scholarships after connecting with NCSA's 42,000 strong network of coaches.

Best 389 Colleges

Best 389 Colleges

165,000 students rate everything from their professors to their campus social scene.

SAT Prep Courses

1400+ course, act prep courses, free sat practice test & events,  1-800-2review, free digital sat prep try our self-paced plus program - for free, get a 14 day trial.

how to write an ap english essay

Free MCAT Practice Test

I already know my score.

how to write an ap english essay

MCAT Self-Paced 14-Day Free Trial

how to write an ap english essay

Enrollment Advisor

1-800-2REVIEW (800-273-8439) ext. 1

1-877-LEARN-30

Mon-Fri 9AM-10PM ET

Sat-Sun 9AM-8PM ET

Student Support

1-800-2REVIEW (800-273-8439) ext. 2

Mon-Fri 9AM-9PM ET

Sat-Sun 8:30AM-5PM ET

Partnerships

  • Teach or Tutor for Us

College Readiness

International

Advertising

Affiliate/Other

  • Enrollment Terms & Conditions
  • Accessibility
  • Cigna Medical Transparency in Coverage

Register Book

Local Offices: Mon-Fri 9AM-6PM

  • SAT Subject Tests

Academic Subjects

  • Social Studies

Find the Right College

  • College Rankings
  • College Advice
  • Applying to College
  • Financial Aid

School & District Partnerships

  • Professional Development
  • Advice Articles
  • Private Tutoring
  • Mobile Apps
  • International Offices
  • Work for Us
  • Affiliate Program
  • Partner with Us
  • Advertise with Us
  • International Partnerships
  • Our Guarantees
  • Accessibility – Canada

Privacy Policy | CA Privacy Notice | Do Not Sell or Share My Personal Information | Your Opt-Out Rights | Terms of Use | Site Map

©2024 TPR Education IP Holdings, LLC. All Rights Reserved. The Princeton Review is not affiliated with Princeton University

TPR Education, LLC (doing business as “The Princeton Review”) is controlled by Primavera Holdings Limited, a firm owned by Chinese nationals with a principal place of business in Hong Kong, China.

AP English Language and Composition Writing Study Skills

Writing is central to the AP English courses and exams. Both courses have two goals: to provide you with opportunities to become skilled, mature, critical readers, and to help you to develop into practiced, logical, clear, and honest writers. In AP English, writing is taught as “process”—that is, thinking, planning, drafting the text, then reviewing, discussing, redrafting, editing, polishing, and finishing it. It’s also important that AP students learn to write “on call” or “on demand.” Learning to write critical or expository essays on call takes time and practice.

Here are some key guidelines to remember in learning to write a critical essay:

  • Take time to organize your ideas.
  • Make pertinent use of the text given to you to analyze.
  • Quote judiciously from the text to support your observations.
  • Be logical in your exposition of ideas.

If you acquire these skills—organizing ideas, marshalling evidence, being logical in analysis, and using the text judiciously—you should have little trouble writing your essays on the AP Exam. Practice in other kinds of writing—narrative, argument, exposition, and personal writing—all have their place alongside practice in writing on demand.

As you study and practice writing, consider the following points.

Reading directly influences writing skills and habits.

Reading and writing are intertwined. When you read what published authors have written you are immersed not just in their ideas, but in the pulsing of their sentences and the aptness of their diction. The more you read, the more that the rhythm of the English language will be available to influence your writing. Reading is not a substitute for writing, but it does help lay the foundation that makes good writing possible.

Writing is fun.

When you have penned what you think is a great sentence or a clean, logical paragraph, read it over to yourself out loud. Enjoy it. Delight in the ideas, savor the diction, and let the phrases and clauses roll around in your mind. Claim it as part of yourself. You may discover you have a voice worthy of respect.

A tip from E. M. Forster

He is reputed to have said that he never knew clearly what it was he thought until he spoke it; and once he had said it, he never knew clearly what it was that he said until he had written it down. Then, Forster noted, he could play with it and give it final form. Be like Forster: think, speak, write, analyze your writing, then give it final shape.

Write purposefully with rhetorical awareness.

When you write, fashion your text with awareness of key rhetorical elements. What is the message of your text? How do you intend to convey your message to your particular audience? Give shape to your thinking with language that enlightens your readers and lets you achieve your aims.

Pay close attention to the task verbs used in the free-response questions. Each one directs you to complete a specific type of response. Here are the task verbs you’ll see on the exam:

  • Analyze: Examine methodically and in detail the structure of the topic of the question for purposes of interpretation and explanation.
  • Argue your position: Formulate a claim and support it with evidence.
  • Read: Look at or view printed directions and provided passages.
  • Synthesize: Combine different perspectives from sources to form a support of a coherent position.
  • Write: Produce a response in writing.

logo-type-white

AP® English Language

How to get a 6 on the argument frq in ap® english language.

  • The Albert Team
  • Last Updated On: March 1, 2022

How to Get a 6 on the Argument FRQ in AP® English Language

What We Review

Introduction: How to Get a 6 on the Argument FRQ in AP® English Language

Wondering how to get a 6 on the argumentative essay in AP® English Language? 

To score an 5 on the AP® English Argument FRQ question, the CollegeBoard scoring guidelines outline that students need to write an essay that effectively argues a position, uses appropriate and convincing evidence, and showcases a wide range of the elements of writing. Essays that score a 6 do all of that and, additionally, demonstrate sophistication in their argument.

An essay that does all of that is an incredibly well-constructed essay. Such an essay needs a solid framework and excellent support. To do this, it is important to have a clear idea of what you are being asked, to not waffle, to spend time and care with your thesis and outline, and to support every claim you make.

We know the best way to write an AP® English FRQ that does everything right is to understand what you are going to see on the AP® English Language test. Read on to prepare yourself for exam day and earn that 6!

What to Expect from the AP® English Language Argument Free Response Questions

The AP® English argument FRQ is the most straightforward of the AP® English FRQs because it is the most similar to the essays you’re already used to writing. It’s exciting to have free reign and make your own argument, unrestrained from rhetorical analysis devices or documents. But, like most AP® writing, it also can be a little overwhelming.

There’s nothing to read and analyze to provide evidence or help you form an argument. Whether you’re feeling excited or overwhelmed by the AP® writing argument FRQ, consider the rhetorical situation. Be strategic about forming your thesis, craft a strong, chronological argument, and utilize good, supportive evidence to earn a better overall essay response.

Determine the question.

The first question to ask yourself is what am I being asked to do ? This may seem obvious, but it’s surprising how tricky it can be to figure out. Look for keywords and phrases that will answer that question.

Here’s an example from the 2019 AP® English Language argumentative essay.

What to Expect from the AP® English Language Argument Free Response Questions - Determine the Question

Though there are just two short paragraphs, there is a lot of room for confusion here. In this case, “Then, write a well-developed essay in which you explain your judgment.” is the key sentence you are looking for. In 2019, AP® English Language test takers were asked to select a concept, place, role, etc. that they believed was “overrated,” and explain why.

If you cannot determine what the question is, go back and reread the prompt. Focus on the last few sentences, as that’s where you’ll usually find it.

Knowing the question you are answering is the most important part of AP® writing. You will not be able to answer the question effectively if you aren’t certain what the question is. Pick out a specific sentence or two to determine the question, and thereby ensure that you aren’t just writing an essay that responds to the general sense of the argument essay prompts

Pick an opinion and stick to it.

The next step is both simple and difficult. Identify your own opinion on the subject.

But remember — the AP® argumentative essay exam format is designed to test how well you can craft an argument. Questions like the 2019 question seem so daunting, because claiming anything to be “overrated” is such a broad topic. It is a bigger question than students are used to encountering on an AP® test.

But, always remember, there is no right or wrong answer for this AP® English FRQ. And whatever argument you choose will not come back later in the exam or in your final grade in the class. This is not to say that you shouldn’t believe in what you are writing. Only that you should remember that both sides are arguable, pick one, and stick to it. Don’t waffle.

Below we break down two sample student answers from this same 2019 prompt. 

What to Expect from the AP® English Language Argument Free Response Questions - PIck an opinion negative example

In this AP® Lang argument essay example, the student jumps from describing places, to people, to outfits. The prompts asked for only one example and the student gives three.  By doing this, it shows they were not only unable to grasp what the prompt was asking, but that they couldn’t stick to their opinion.  Instead of deeply strengthening one choice, the student gives vague, half-reasons for too many choices. When writing your FRQs, choose just one concept and stick to it.

The following example demonstrates a strong student response:

What to Expect from the AP® English Language Argument Free Response Questions - Pick an opinion strong example

This student picks one clear concept, capitalism, and clearly outlines their support for it.  They write with clear language that opens the door for the deeper analysis coming later in the essay.

Like this student, choose just one clear argument to delve into when writing your FRQ.

Craft a thesis statement.

The thesis statement should be both simple and elegant. Students often find it one of the more difficult writing skills to master, but we’re here to help. Just remember that it should encompass your entire essay in just one sentence.  So, for the 2019 argument FRQ :

Good thesis: While capitalism undeniably has its upsides, it has many downsides that are rarely recognized. When considering the downsides, capitalism is clearly overrated as it commodifies humanity and uplifts a minority at the expense of the majority.

This thesis breaks down a) that the author clearly states his claim that capitalism is overrated, b) that the author will support that claim with examples on how it commodifies humanity and how it hurts the majority in favor of the minority.

Good thesis: While the Electoral College was created in the name of equality for smaller states, it is ultimately overrated because it undercuts the popular vote, it is an archaic practice that is unsuitable for the modern era.

This thesis claims the Electoral College is overrated by claiming it doesn’t do what it was created to do in the first place- support equality.  It also introduces two supporting examples for the rest of the essay- it undercuts the popular vote and it doesn’t work in the modern era.

Not a good thesis: Kicking a ball in a net and scoring, is not as important as saving lives. Soccer to me would be considered overrated.

This thesis doesn’t give clear direction for the rest of the essay.  The author claims soccer is overrated, but doesn’t tell us why. The example that “it’s not as important as saving lives” is unrelated and also not touched on again later in the essay. This thesis isn’t specific and doesn’t give you a clear idea of what the author will be saying next.

Not a good thesis: The term “overrated” has been used in conversation to diminish the value of roles. In unusual circumstances the term “overrated” should be applied to the idea of freedom in regards to social change, but overall it should not be applied in regards to global devastation and cruel treatment.

This thesis does not directly answer the question.  Is the author arguing that freedom is overrated? They also claim that the term overrated doesn’t apply to global devastation and cruel treatment. This second claim is both unrelated to the first and doesn’t work to answer the initial prompt.

Looking at these four examples, can you see the difference between a strong and weak thesis?

After you’ve determined your thesis, use it as a jumping point to sketch a quick outline. Then, follow your outline, bringing in your own concrete examples and evidence. Doing so will improve your AP® writing.

Return to the Table of Contents

Craft a chronological argument.

A good argument builds as you move through the essay. It does not simply repeat the same points. Instead, the different points of the argument build off one another and work together to advance the author’s point.

Let’s look at the 2018 AP® English argument FRQ for an example.

What to Expect from the AP® English Language Argument Free Response Questions - Craft a chronological argument

In this case, students are being asked to argue a position on the value of choosing the unknown. 

All students are likely to have their own definitions of what “choosing the unknown” might mean. You first want to consider what this phrase means to you, and how it applies to the real world.  Could it mean breaking out of your comfort zone in daily routines, or could it mean going to theater school to follow your dreams?  There’s no wrong answers, but try to pin down one. Consider Lindbergh’s quote the prompt gives you, and how shock, disappointment, and enrichment play into choosing the unknown.

Once you’ve nailed down your definition, you can begin to form your arguments. A chronological argument builds off itself. So, in this question’s case, an outline would look something like this:

  • Choosing the unknown is necessary for the development of the human race.
  • Scientific advancements cannot be made without testing the boundaries of the unknown.
  • Cultural and artistic growth can only occur through exploring the unknown.

First, a student must define what choosing the unknown means, and what makes it difficult. Next the student argues for the value of choosing the unknown, in that the human race could never develop without it.  Finally, the student will argue for the invaluable scientific and cultural/artistic advances made throughout history by breaking known boundaries.

When you sketch your outline, quickly ask yourself if the outline would make just as much sense if you rearranged it. If the answer is no, start writing your essay. If the answer is yes, try to structure your argument so that your points build off one another.

Support your claims.

All arguments need evidence. This is the proof you need to support your thesis. And in the case of the AP® English argument FRQ, the evidence all comes from you. What exactly that evidence is will vary from question to question and from student to student. But make sure that every point you make is supported by evidence.

Here’s some good news — you already know quite a bit about effective evidence from what you have learned in AP® English about rhetorical devices. Your main purpose in this essay is to persuade. What have you learned in class about effective ways to persuade? What rhetorical devices can you utilize? Try to pick the best devices to support your argument that you can.

Here are some examples of supportive and non-supportive evidence that students could use to support their claims.

What to Expect from the AP® English Language Argument Free Response Questions - Support your claims

The 2017 AP® English language argument FRQ asked students to argue a position if the most essential skill is artifice. The example student answers given below are from here .

Supportive evidence:   “Throughout history, rulers have utilized countless different methods of achieving power, however none have been so successful as mastering the art of lying.

In his advice to future rulers, Niccolo Machiavelli encouraged them to lie and maintain the illusion of sympathy to the common struggles in order to retain power. He asserts that it is imperative for a ruler to appear caring and sympathetic even if he has no objective but power.

Machiavelli argues that to be sincere and honest is akin to being vulnerable. A ruler must be skilled in the art of deception if he is not to fall prey to usurpers. Thus, it is essential that he appear humble and morally upright to his constituents as he is to appear idealistic, despite his nature being identical to his citizens.”

In this paragraph, the student chooses to discuss the role of artifice in politics. The student claims that mastering lying is essential to achieving political power. The student uses Machiavelli’s leadership and beliefs as specific examples to support this, by analyzing and connecting each point back to his/her claim.

Non-supportive evidence: “Another example would be actors on red carpets or at interviews they sound generous and relatable, but in reality they could be selfish people who don’t care about anyone. To the public they act charming, honest, and sincere. They do this so they can get famous and rich. They do this so they will never get ignored.”

In this paragraph, the student chooses to discuss the role of artifice in the culture of entertainment and celebrities. However, the student does not utilize supportive evidence to do so. The paragraph is full of claims about how actors lie, but does not provide a concrete example to anchor the claims. The student provides a lot of very vague generalizations, but no clear evidence or examples of specific celebrities and how they used artifice to succeed.

There is so much variance in prompts and students’ prior knowledge; it’s impossible to provide a checklist of what makes evidence supportive. But a good trick to decide if you’ve supported your claims well enough is to talk to yourself. No really, it’s a good idea.

Picture yourself discussing your essay with someone. Imagine that this person disagrees with everything that you say. Every time you make a claim, like that it’s important to be polite in an email, your imaginary person shakes their head and tells you no. How would you try to convince them? What examples would you use? Make sure that for each opinion you put forward; you have provided an answer to someone who would disagree with you.

The evidence is an important part of your essay. If your outline and your argument are a framework, your evidence is the brick and mortar. A house without brick and mortar won’t fall, but it won’t be a very nice house to inhabit. Tie every claim you make to a piece of evidence to ensure the best essay possible.

Wrapping Things Up: Scoring a 6 on the Argument FRQ for AP® English Language

The AP® English argument FRQ varies quite a bit. But it is ultimately about how well you can put forth an argument. So, don’t be afraid to spend some time crafting that argument. We’ve covered a lot in this article- here are the main points to remember:

  • Determine the question. Figure out what the prompt is asking you to do.
  • Pick an opinion and stick to it. Choose one side of the argument and one clear claim to support all the way through.
  • Craft a thesis statement. Your thesis should be clear, concise, and introduce the content of your essay.
  • Craft a chronological argument. Make an argument that builds on its prior points.
  • Support your claims. Support yourself with concrete, specific evidence and examples. 

But most of all, have fun. This essay is the one you should be looking forward to, where you have the freest rein. Enjoy it and earn yourself a 6.

Do the examples shown make sense to you? Can you picture yourself moving through the AP® writing argument FRQ with ease now?

Interested in a school license?​

8 thoughts on “how to get a 6 on the argument frq in ap® english language”.

Thank you for explaining this so eloquently. Excellent post, I will keep this handy and refer to it often from now on. It’s so educative. Great post!

Sure, glad it helped.

I’m an AP® Language teacher and the title of your article caught my eye because the essays aren’t scored on a 0-9 scale anymore. The max score for an essay now is a 6. Essays are now scored in 3 categories: Thesis: 0 or 1 point Evidence and commentary: 0-4 points Sophistication: 0 or 1 point I just wanted to let you know! I saw this was last updated in 2020 and just thought it should reflect the current AP® exam.

Thank you for the heads up! This is an older blog post that must have had something else updated to it this year. We’ve gone ahead and revised the post.

Hi, my AP® Language teacher emphasized on a counterargument at the end of the supporting paragraphs. Could you elaborate on it? Also, how exactly do we get the sophistication point?

Hi Stephanie, thanks for reaching out! Making a solid counter-argument is definitely one way to make sure that you earn the Sophistication point. We recommend having a look at our AP® English Language Review Guide for 2021 for more tips! The College Board’s Free-Response Question and Scoring Information Archive also provides authentic examples of student writing — many of which successfully make counterarguments and rebuttals to earn the Sophistication point.

Hi can I get a 6?

Hi Roy, we certainly believe that earning a 6 on your FRQs is possible with practice and dedication! I’d recommend having a look at our AP® English Language Review Guide for tips and tricks, and you can also browse our AP® English Language and Composition Resource Page and Free Response practice questions for targeted practice.

Comments are closed.

Popular Posts

AP® Physics I score calculator

AP® Score Calculators

Simulate how different MCQ and FRQ scores translate into AP® scores

how to write an ap english essay

AP® Review Guides

The ultimate review guides for AP® subjects to help you plan and structure your prep.

how to write an ap english essay

Core Subject Review Guides

Review the most important topics in Physics and Algebra 1 .

how to write an ap english essay

SAT® Score Calculator

See how scores on each section impacts your overall SAT® score

how to write an ap english essay

ACT® Score Calculator

See how scores on each section impacts your overall ACT® score

how to write an ap english essay

Grammar Review Hub

Comprehensive review of grammar skills

how to write an ap english essay

AP® Posters

Download updated posters summarizing the main topics and structure for each AP® exam.

What are your chances of acceptance?

Calculate for all schools, your chance of acceptance.

Duke University

Your chancing factors

Extracurriculars.

how to write an ap english essay

Ultimate Guide to the AP English Language and Composition Exam

Do you know how to improve your profile for college applications.

See how your profile ranks among thousands of other students using CollegeVine. Calculate your chances at your dream schools and learn what areas you need to improve right now — it only takes 3 minutes and it's 100% free.

The AP Language and Composition exam is one of the most popular exams taken year after year. In fact, in 2019, over 10% of the more than five million students who took AP exams took the Language and Composition test. AP Lang is most often taken by high school juniors, many of whom go on to take the AP English Literature exam their senior year. Plenty of seniors and even sophomores take this test too though, contributing to its popularity. If you’re planning to take the AP Language and Composition exam, whether you have taken the class or self-studied, look no further. Here’s our complete guide to the AP Lang exam, full of expert tips and free study resources.

When is the AP Language and Composition Exam?

On Wednesday, May 6, at 8 am, the College Board will hold the 2020 AP Language and Composition exam. For a comprehensive listing of all the AP exam times and AP score distributions from 2019, check out our post 2020 AP Exam Schedule: Everything You Need to Know .    

About the AP Language and Composition Exam

The AP Language and Composition exam is based primarily on the study of rhetoric, wherein an author attempts to persuade, inform, or motivate an audience using established techniques. The College Board encourages students who are interested in studying and writing various kinds of analytic or persuasive essays on nonliterary topics to take this course. It tests students on their reading comprehension, rhetorical analysis, synthesis of information, and written argumentation.

Big Ideas: The AP Language and Composition exam is built on a foundation of four big ideas. Big ideas are threads that run throughout the AP Language and Composition course that are vital for making connections and developing a deeper understanding of concepts found within it. The four big ideas are: 

  • Rhetorical Situation: Understanding what an author is communicating, how they convey that message, and what the impact of their rhetorical strategies are.
  • Claims and Evidence: Making claims and justifying them, while acknowledging or responding to opposing arguments. 
  • Reasoning and Organization: Guiding a reader’s understanding of text through its organization and the development of its argument. 
  • Style: The stylistic choices writers make and their impact. 

Course Skills: Along with exploring and connecting concepts with big ideas, students will develop eight course skills—four sets of two paired reading and writing skills—necessary for analyzing and composing arguments. The course skills and the weight they’re given on the multiple-choice section of the AP Language and Composition exam are: 

Rhetorical Situation – Reading Explain how writers’ choices reflect the components of the rhetorical situation. 11%-14%
Rhetorical Situation – Writing Make strategic choices in a text to address a rhetorical situation. 11%-14%
Claims and Evidence – Reading Identify and describe the claims and evidence of an argument. 13%-16%
Claims and Evidence – Writing Analyze and select evidence to develop and refine a claim. 11%-14%
Reasoning and Organization – Reading Describe the reasoning, organization, and development of an argument. 13%-16%
Reasoning and Organization – Writing Use organization and commentary to illuminate the line of reasoning in an argument. 11%-14%
Style – Reading Explain how writers’ stylistic choices contribute to the  purpose

of an argument.

11%-14%
Style – Writing Select words and use elements of composition to advance an argument. 11%-14%

  About the AP Language and Composition Exam Content

The Language and Composition exam is one of the longer AP exams, clocking in at 3 hours and 15 minutes from start to finish. The Language and Composition exam is structured in two sections—one featuring multiple-choice, the other free-response questions. 

Section 1: Multiple Choice 

1 hour | 45 questions | 45% of score

There have been some changes to the AP Language and Composition Exam for 2020. The first section remains one hour long and is still worth 45% of your score, but the number of questions has shrunk from 52-55 to 45. The variance in types of questions asked is also now more clearly defined—questions are now presented in 5 sets with 23-25 reading questions (reading and analyzing nonfiction texts) and 20-22 writing questions (“read like a writer” and consider revisions to stimulus texts), both of which will use shorter stimuli than previous exams. Below is the structure of the multiple-choice section of the AP Language and Composition exam. 

1 11-14 Reading Skills
2 11-14 Reading Skills
3 7-9 Writing Skills
4 7-9 Writing Skills
5 4-6 Writing Skills

Sample of a multiple-choice reading question: 

ap lang sample questions

Sample multiple-choice writing question: 

ap lang sample question

Section 2: Free Response

2 hour 15 minutes | 3 questions | 55% of score

The second section takes 2 hours and 15 minutes to complete and consists of 3 free response questions worth 55% of your score. These prompts are each of a different type: one synthesis question, one passage analysis, and one argumentative essay.

Synthesis Question: The synthesis question asks students to consider a scenario and then formulate a response to a specific element of it using at least three accompanying sources for support. Sources used in the essay need to be cited to be considered legitimate.  

Sample synthesis free response question: 

ap lang sample question

Analysis Question: The analysis question asks students to read a short passage and analyze and discuss various devices used by the author, such as strategies, argumentative techniques, or motivations. 

Sample analysis free response question: 

ap lang sample question

Argument Question: The argument question gives a position in the form of an assertion from a documented source and asks students to form their own argument to defend, challenge, or qualify it using supporting evidence. 

Sample argument free response question: 

ap lang sample question

The format of the free response section is unchanged this year; however, the scoring has shifted from a holistic rubric to an analytic rubric. The new rubric hasn’t been released, but you can gain insight into what type of answers the College Board is looking for by reading the sample free response questions found in the AP Language and Composition Course and Exam Description . 

AP Language and Composition Score Distribution, Average Score, and Passing Rate

AP Language and Composition    9.9% 18.2% 26.2% 31.2% 14.5%

In 2019, 54.3% of the students who took the AP English Language and Composition exam received a score of 3 or higher. Only 9.9% of students who took the exam achieved the top score of 5, and 14.5% of students who took the exam scored a 1. That said, students take the course seriously and prepare diligently will often find that the test is not as difficult as the results indicate.

 If you’re curious about other score distributions, see our post Easiest and Hardest AP Exams .

how to write an ap english essay

Best Ways to Study for the AP Language and Composition Exam

Step 1: assess your skills.

Take a practice test to assess your initial knowledge. Though the College Board AP Language and Composition website provides a number of sample test questions, it does not provide a complete sample test. You can find a practice test in many of the official study guides, and some even include a diagnostic test to act as your initial assessment. Varsity Tutors offers a handful of free AP Language and Composition diagnostic tests on its website. You’ll also find a free practice exam from College Countdown to use for your assessment. 

Once you have taken some kind of formative assessment, score it to identify your areas of strength and areas in need of improvement. It can be helpful to have a teacher or friend score your free-response essays, as these are a bit more subjective than the multiple-choice section. With an accurate formative assessment, you’ll have a better idea of where to focus your studying efforts.

Step 2: Know Your Material

In the case of AP English Language and Composition, this means focusing on your reading and writing skills.

When reading, make sure to preview important elements such as the title, author’s name, and any other information available like the table of contents or introduction. As you read, make sure to stop periodically to consider the main ideas and the way the author supports them. Underline important evidence as you go. Reread complex or important sentences.      

One consultant to the College Board writes about the “SOAPSTone” approach to reading, which is an acronym for a series of questions that students should ask themselves when analyzing a 

piece of prose. The questions are:

  • Who is the Speaker?
  • What is the Occasion?
  • Who is the Audience?
  • What is the Purpose?
  • What is the Subject?
  • What is the Tone?

For more about using this technique, read about it on the College Board website .

Writing high-quality free-response essays takes practice and time. Make sure to organize your ideas using a rough outline before you begin writing. Use direct evidence from the text to support your ideas, and quote judiciously with correct citations. As you’re writing, be aware of rhetorical elements and use them effectively.

For more specific information about the test, consider using a formal study guide, such as Barron’s AP English Language and Composition, 7th Edition or the Princeton Review’s Cracking the AP English Language & Composition Exam 2020, Premium Edition . 

Alternatively, there are many online study resources available. Some AP teachers have even published their own study guides or review sheets online, like this AP Lang guide by Mrs. Smith at Pinnacle High School.

Another way to study is to use one of the recently-developed apps for AP exams. These are a great way to get practice questions in while on-the-go. Make sure you read reviews before choosing one, as their quality varies widely. This AP Lang app by Varsity Tutors has decent reviews, and might be worth checking out.

Step 3: Practice Multiple-Choice Questions

Once you have your theory down, test it out by practicing multiple-choice questions. You can find these in most study guides or through online searches. Here are some AP Lang practice questions and tests , and more are available in the College Board’s course description .

Try to keep track of which areas are still tripping you up, and go back over this theory again. Keep in mind, the key to answering questions correctly is understanding the passage, so practice active reading skills as you tackle the multiple-choice questions. This includes underlining, mouthing words, and circling key points. Remember, the answer will always be found in the text.

Step 4: Practice Free-Response Essays

As indicated on your exam, it is recommended that you spend 15 minutes reading the question, analyzing, and evaluating the sources, and 40 minutes writing your response. Try to stick to this timeline when practicing your free-response essays to see if it works for you. You do not have to follow it on exam day, but having a good idea of how much time it typically takes for you to plan and write will be an advantage.

As you tackle your open responses, identify what each is asking you to do. When asked to synthesize, you know you will be taking pieces of evidence from multiple sources to form a single argument. Use specific examples and make them stand out by explicitly stating, “For example…” or “As Source C indicates in paragraph 3…” Also, be sure to cite your sources appropriately while writing.

When writing an analysis of rhetorical strategies used, first consider the elements of SOAPSTone as discussed above. Also consider the five canons of rhetoric . This means thinking about the author’s invention, arrangement, and style. Memory and delivery are obviously less apparent in written pieces, but their roles in a speech are still important. As you read, try to underline specific places that highlight relevant examples.   

Finally, when writing your own persuasive argument, support your ideas with concrete examples from current events, literature, etc. Try to vary your sources to build credibility and address counterpoints to craft an even stronger response.

As you prepare for the writing portion of your exam, be sure to review how your free responses will be scored. The College Board supplies free response questions and authentic scored student responses with written explanations dating back to 1999; these are an invaluable tool for this exercise.

Step 5: Take Another Practice Test

Take another practice test to evaluate the progression of your knowledge, as well as identify persistent areas of weakness. Study.com offers a free online practice AP Language and Composition exam . Over time, you should begin to notice areas in which your studying should be increased and those which you are strong in. Repeat the above steps if time permits to incrementally increase your score. 

Step 6: Exam Day Specifics

If you’re taking the AP course associated with this exam, your teacher will walk you through how to register. If you’re self-studying, check out our blog post How to Self-Register for AP Exams .

For information about what to bring to the exam, see our post What Should I Bring to My AP Exam (And What Should I Definitely Leave at Home)?

Want access to expert college guidance — for free? When you create your free CollegeVine account, you will find out your real admissions chances, build a best-fit school list, learn how to improve your profile, and get your questions answered by experts and peers—all for free. Sign up for your CollegeVine account today to get a boost on your college journey.

For more guidance about the AP exams, check out these other informative articles: 

2020 AP Exam Schedule

How Long is Each AP Exam?

Easiest and Hardest AP Exams

Related CollegeVine Blog Posts

how to write an ap english essay

Get the Reddit app

No matter what course you are taking, we are a community that helps students earn college credit!

How to self study for English Language and Composition Test

Hi everyone,

This is my first post here, I am a junior and my school requires our entire grade to take the AP exam for English lang and comp, most people don't care how they do, but I really want to do well. For those of you who have taken the test how do you recommend that I study since I am just taking the test and not the actual test?

By continuing, you agree to our User Agreement and acknowledge that you understand the Privacy Policy .

Enter the 6-digit code from your authenticator app

You’ve set up two-factor authentication for this account.

Enter a 6-digit backup code

Create your username and password.

Reddit is anonymous, so your username is what you’ll go by here. Choose wisely—because once you get a name, you can’t change it.

Reset your password

Enter your email address or username and we’ll send you a link to reset your password

Check your inbox

An email with a link to reset your password was sent to the email address associated with your account

Choose a Reddit account to continue

Skip to Content

Other ways to search:

  • Events Calendar

Want to write a college essay that sets you apart? Three tips to give you a head start

How to write a college essay

1. Keep it real. It’s normal to want to make a good impression on the school of your choice, but it’s also important to show who you really are. So just be yourself! Compelling stories might not be perfectly linear or have a happy ending, and that’s OK. It’s best to be authentic instead of telling schools what you think they want to hear.

2. Be reflective . Think about how you’ve changed during high school. How have you grown and improved? What makes you feel ready for college, and how do you hope to contribute to the campus community and society at large?

3. Look to the future. Consider your reasons for attending college. What do you hope to gain from your education? What about college excites you the most, and what would you like to do after you graduate? Answering these questions will not only give colleges insight into the kind of student you’ll be, but it will also give you the personal insight you’ll need to choose the school that’s right for you.

Have questions about college prep? We're here to help.

Written by CU Boulder Office of Admissions

  • College-Prep

The University of Colorado does not discriminate on the basis of race, color, national origin, sex, age, pregnancy, disability, creed, religion, sexual orientation, gender identity, gender expression, veteran status, political affiliation, or political philosophy. All qualified individuals are encouraged to apply. You may  view the list of ADA and Title IX coordinators  and  review the Regent policy .

As a student or prospective student at CU Boulder, you have a right to certain information pertaining to financial aid programs, the Clery Act, crime and safety, graduation rates, athletics and other general information such as the costs associated with attending CU Boulder. To view this information visit  colorado.edu/your-right-know .

Apply for Admission

Visit Campus

Support CU Boulder

  • Safety & Health Services
  • COVID-19 Information
  • Campus Communications
  • Emergency Alert System
  • New Student & Family Programs

Getting Around

  • Campus Events
  • Parking & Transportation
  • Visit Information

Information for

  • Faculty & Staff
  • Journalists

Initiatives

  • Business & Industry Collaborations
  • Diversity, Equity & Inclusion
  • Free Speech
  • Innovation & Entrepreneurship
  • Public & Outreach Programs
  • Sustainability
  • Understanding Your Cost of Attendance

IMAGES

  1. AP Language and Composition argumentative essay

    how to write an ap english essay

  2. ⭐ Ap english language and composition rhetorical analysis essay sample

    how to write an ap english essay

  3. Ap Lang Analysis Essay Examples

    how to write an ap english essay

  4. Ultimate Guide to the AP English Language and Composition Exam

    how to write an ap english essay

  5. AP Lang Argument Essay Outline by Walden's Wonderland

    how to write an ap english essay

  6. How to Write a Synthesis Essay

    how to write an ap english essay

COMMENTS

  1. How to Write the AP Lang Argument Essay + Examples

    2. Pick one side of the argument, but acknowledge the other side. When you write the essay, it's best if you pick one side of the debate and stick with it for the entire essay. All your evidence should be in support of that one side. However, in your introductory paragraph, as you introduce the debate, be sure to mention any merit the ...

  2. How to Write the AP Lang Synthesis Essay + Example

    Step 5: Write your Essay. Use the remaining 30-35 minutes to write your essay. This should be relatively easy if you took the time to mark up the sources and have a detailed outline. Remember to add special consideration and emphasis to the commentary sections of the supporting arguments outlined in your thesis.

  3. How to Write the AP Lang Rhetorical Analysis Essay (With Example)

    The AP Lang Rhetorical Analysis Essay is one of three essays included in the written portion of the AP English Exam. The full AP English Exam is 3 hours and 15 minutes long, with the first 60 minutes dedicated to multiple-choice questions. Once you complete the multiple-choice section, you move on to three equally weighted essays that ask you ...

  4. How to Write a Perfect Synthesis Essay for the AP Language Test

    Step 5: Draft Your Essay Response. The great thing about taking a few minutes to develop an outline is that you can develop it out into your essay draft. After you take about 5 to 10 minutes to outline your synthesis essay, you can use the remaining 30 to 35 minutes to draft your essay and review it.

  5. How to Write the AP Lang Rhetorical Essay

    4. Be Sure to Explain Your Examples. As you write the essay, don't just list out your examples and say something like "this is an example of ethos, logos, pathos.". Instead, analyze how the example shows that rhetoric device and how it helps the author further their argument. As you write the rhetorical essay, you'll want to be as ...

  6. How to Write the AP Lang Argument Essay (With Example)

    Write an essay that argues your position on the value of striving for perfection. In your response you should do the following: Respond to the prompt with a thesis that presents a defensible position. Provide evidence to support your line of reasoning. Explain how the evidence supports your line of reasoning.

  7. AP English Language and Composition: Pace Your Essay Writing

    With an average time of only 40 minutes per essay for your AP English Language and Composition exam, you should divide your time as follows. Spend about 10 minutes reading the topic and the passage carefully and planning your essay. This organizational time is crucial to producing a high-scoring essay. Consider following these steps:

  8. How to Write the AP Lang Synthesis Essay with Example

    AP Lang Exam Basics. The AP Lang exam is separated into two sections. In the first section, students have one hour to answer a series of 45 multiple-choice questions. Here, about half of the questions are based on passages students read. The other half are focused on the best revision techniques.

  9. Acing the AP® English Language and Composition Synthesis Essay

    The newest section of the AP® English Language and Composition Exam, the synthesis essay, is one of three essays you will be completing during the examination's 2-hour free-response period. However, you'll also have a 15-minute reading and planning period just for this essay, and if you use this time to plan effectively, you can't go wrong.

  10. Tackling the AP English Language and Composition essays: part 1

    Writing Part 1: Paragraphs (Intro, Evidence, Analysis, Conclusion) Writing Part 2: Sentence by Sentence #1 Organizing your time. On the AP Lang exam, you get a total of 2 hours and 15 minutes to write your three essays. This time is split into chunks. First, there is a 15 minute "reading period"; next, there is a 2 hour "writing period."

  11. Expert's Guide to the AP Literature Exam · PrepScholar

    The AP Literature Exam is a three-hour exam that contains two sections in this order: An hour-long, 55-question multiple-choice section. A two-hour, three-question free-response section. The exam tests your ability to analyze works and excerpts of literature and cogently communicate that analysis in essay form.

  12. How to Ace the AP Language Rhetorical Analysis Essay

    In this video, I'll show you how to write the AP English Language rhetorical analysis essay (Q2) step by step using the actual 2017 prompt. Watch me annotate...

  13. Expert Guide to the AP Language and Composition Exam

    The AP English Language and Composition Multiple-Choice. The multiple-choice section tests you on two main areas. The first is how well you can read and understand nonfiction passages for their use of rhetorical devices and tools. The second is how well you can "think like a writer" and make revisions to texts in composition questions.

  14. AP English Language and Composition

    Starting in the 2024-25 school year, AP English Language and Composition multiple-choice questions (MCQs) will have four answer choices instead of five. This change will take effect with the 2025 exam. ... Students write essays that respond to 3 free-response prompts from the following categories: Synthesis Question: ...

  15. Tackling the AP English Language and Composition Essays: Part 3

    6. Writing: Sentence by Sentence. Proofread, at the end! The same tips from Part 2 apply. Well, that just about covers the three AP Lang Essays! I have just two parting tips. First, practice, practice, practice. It will not feel easy writing three good essays at this pace the first time you do it (or the second, or the third).

  16. Guide to the AP English Language and Composition Exam

    The three Writing passages are student-produced essays. The idea is to get you to revise the essay that help the writer accomplish his or her goal. Free Response . The AP English Language section contains three essay prompts: a synthesis essay, a rhetorical analysis essay, and an argument essay.

  17. How to Craft an Argument for AP® English Language

    Persuasion through essay writing is something you probably learned about a long time ago, but the AP® English Language Exam's persuasion essay requires some more specific tips. You will be given a prompt that may or may not reference a reading sample; it will ask you to then "defend," "challenge," or "qualify" a position on a ...

  18. AP English Literature and Composition Writing Study Skills

    In AP English, writing is taught as "process"—that is, thinking, planning, drafting the text, then reviewing, discussing, redrafting, editing, polishing, and finishing it. It's also important that AP students learn to write "on call" or "on demand.". Learning to write critical or expository essays on call takes time and practice.

  19. AP English Language and Composition Writing Study Skills

    In AP English, writing is taught as "process"—that is, thinking, planning, drafting the text, then reviewing, discussing, redrafting, editing, polishing, and finishing it. It's also important that AP students learn to write "on call" or "on demand." Learning to write critical or expository essays on call takes time and practice.

  20. PDF AP English Language and Composition

    Write an essay that argues your position on the value of striving for perfection. In your response you should do the following: • Respond to the prompt with a thesis that presents a defensible position. • Provide evidence to support your line of reasoning. • Explain how the evidence supports your line of reasoning.

  21. How to Get a 6 on Argument FRQ in AP® English Language

    Pick an opinion and stick to it. Choose one side of the argument and one clear claim to support all the way through. Craft a thesis statement. Your thesis should be clear, concise, and introduce the content of your essay. Craft a chronological argument. Make an argument that builds on its prior points.

  22. Ultimate Guide to the AP English Language and Composition Exam

    AP Language and Composition Score Distribution, Average Score, and Passing Rate. In 2019, 54.3% of the students who took the AP English Language and Composition exam received a score of 3 or higher. Only 9.9% of students who took the exam achieved the top score of 5, and 14.5% of students who took the exam scored a 1.

  23. How to self study for English Language and Composition Test

    I would recommend going through as many essays as possible leading up to the date of the exam, either writing the essays in full or writing quick yet detailed outlines for each. Read A LOT. Multiple choice is going to be the hardest part of the test to study for, particularly because the MCQ portion was redesigned in 2020 and there are very few ...

  24. Want to write a college essay that sets you apart? Three tips to give

    Writing the personal essay for your college application can be tough, but we're here to help. Sometimes the hardest part is just getting started, but the sooner you begin, the more time and thought you can put into an essay that stands out. Check out some tips: 1. Keep it real.